Une intégrale de plus

Bonjour à tous.
Avez-vous quelques idées pour montrer que (Maple le dit !) $\displaystyle\int_0^{+\infty}\frac{\ln(x)}{1+\mathrm{e}^x}\mathrm{d}x=-\frac{\ln^2(2)}2$ ?
J'ai tenté d'introduire une intégrale à paramètre, faire apparaître l'intégrale en fonction d'elle-même (le facteur $\frac 12$...), faire des IPP/changements de variables, viser $\frac{\ln(1+x)}{1+x}$ pour viser le $\ln^2$, développer en série entière via le développement de $\frac 1{1+x}$... en vain ! Voyez-vous un moyen "élémentaire" de conclure ? Merci !

Réponses

  • Bonjour
    De mémoire on peut demander à Maple de "tracer" son calcul (printlevel). Cela peut aider!
  • Merci vincent83 et Fdp...j'étais assez loin du compte !
  • Dedekind93:

    Formellement:
    $\displaystyle\int_0^{+\infty}\frac{\ln(x)}{1+\mathrm{e}^x}\mathrm{d}x=\int_0^\infty \frac{\mathrm{e}^{-x}\ln x}{1+\mathrm{e}^{-x}}dx=\int_0^{\infty}\left(\sum_{n=0}^{\infty} (-1)^n\mathrm{e}^{-(n+1)x}\right)\ln x dx= \sum_{n=0}^{\infty}\int_0^\infty (-1)^n\mathrm{e}^{-(n+1)x}\ln x dx$

    Cela doit exister une formule générale pour calculer l'intégrale. Ce que je sais est que $\displaystyle \int_0^\infty \mathrm{e}^{-x}\ln x dx=-\gamma$

    PS:
    Il se peut aussi que l'interversion du signe somme et du signe intégrale ne soit pas licite.

    PS2:
    J'avais oublié $(-1)^n$ dans la somme. Merci Chaurien.
  • Il y a un facteur $(-1)^{n}$ qui a été oublié dans la série géométrique. Ainsi, on obtient une série alternée convergente.
  • Après élimination de $\gamma$, je trouve bien $- \frac12 (\ln 2)^2$. J'ai d'abord refait les calculs parce que je n'avais pas lu le début du fil (toujours mon étourderie !) et je m'étonnais de trouver une valeur négative, mais c'était idiot.
  • La méthode de FdP conduit bien au résultat, au moyen de la série $\displaystyle \overset{+\infty }{\underset{n=1}{\sum }}(-1)^{n}\frac{\ln n}{n}=\gamma \ln 2-\frac{1}{2}(\ln 2)^{2}$, qui est assez connue. Parue je ne sais plus quand dans l'AMM, elle a été posée à des oraux de concours, et je l'ai dans mes papiers depuis longtemps.
    Mais l'apparition-disparition de la constante d'Euler peut nous faire penser qu'il y aurait peut-être une autre solution.
  • Notons $$
    F(z) := \int_0^{+\infty}\frac{t^{z-1}}{1+\mathrm{e}^t}\mathrm{d}t = (1-2^{1-z})\Gamma(z)\zeta(z), \qquad \Re z >0.
    $$ Ainsi, $$
    F'(z) = \int_0^{+\infty}\frac{t^{z-1} \ln t}{1+\mathrm{e}^t}\mathrm{d}t = 2^{1-z} \ln2 \Gamma(z)\zeta(z) + (1-2^{1-z})\Gamma'(z)\zeta(z) + (1-2^{1-z})\Gamma(z)\zeta'(z).
    $$
  • L'intégrale demandée donc est :
    $$ \int_0^{+\infty}\frac{\ln t}{1+\mathrm{e}^t}\mathrm{d}t = \lim_{z \to 1} \left( 2^{1-z} \ln2 \Gamma(z)\zeta(z) + (1-2^{1-z})\Gamma'(z)\zeta(z) + (1-2^{1-z})\Gamma(z)\zeta'(z) \right).$$
  • A savoir :
    $ \Gamma(1)=1 \quad;\quad \Gamma'(1)=-\gamma $

    Les développements en série de Laurent des fonctions $\zeta$ et $\zeta'$ (wikipedia.org) :

    $$\begin{align} &\zeta(z) = \frac{1}{z-1} + \sum_{n=0}^{+\infty} \frac{(-1)^n \gamma_n}{n!}(z-1)^n = \frac{1}{z-1} + \gamma + o(z-1) \quad;\quad \gamma_0=\gamma.\\
    &\zeta'(z) = - \frac{1}{(z-1)^2} + \sum_{n=1}^{+\infty} \frac{n(-1)^n \gamma_n}{n!}(z-1)^{n-1} = - \frac{1}{(z-1)^2} - \gamma_1 + o(z-1) \\
    \end{align}$$
  • Ce problème a été posé dans The American Mathematical Monthly, Problem 4394, Vol. 57, No. 4, April 1950, par Henry Francis Sandham, une vieille connaissance : http://www.les-mathematiques.net/phorum/read.php?4,2260908,2261214#msg-2261214.
    Solution dans The American Mathematical Monthly, Vol. 58, No. 10, Dec.1951, pp. 705-706. Je la joins.
    Bonne nuit.
    Fr. Ch.
  • Récapitulons :
    $$ \int_0^{+\infty}\frac{\ln t}{1+\mathrm{e}^t}\mathrm{d}t = \lim_{z \to 1} \left( 2^{1-z} \ln2 \Gamma(z)\zeta(z) + (1-2^{1-z})\Gamma'(z)\zeta(z) + (1-2^{1-z})\Gamma(z)\zeta'(z) \right).
    $$ $ \Gamma(1)=1 \quad;\quad \Gamma'(1)=-\gamma $

    Les développements en série de Laurent des fonctions $\zeta$, $\zeta'$ et $z \mapsto 1-2^{1-z}$ (wikipedia.org) :
    $$
    \begin{align} &\zeta(z) = \frac{1}{z-1} + \sum_{n=0}^{+\infty} \frac{(-1)^n \gamma_n}{n!}(z-1)^n = \frac{1}{z-1} + \gamma + o(z-1) \quad;\quad \gamma_0=\gamma.\\
    &\zeta'(z) = - \frac{1}{(z-1)^2} + \sum_{n=1}^{+\infty} \frac{n(-1)^n \gamma_n}{n!}(z-1)^{n-1} = - \frac{1}{(z-1)^2} - \gamma_1 + o(z-1) \\
    &1-2^{1-z} = (z-1)\ln2 - \frac{\ln^22}{2}(z-1) ^2 + o\big((z-1)^3\big) .
    \end{align}
    $$ Ainsi, $$\begin{align} 2^{1-z} \ln2 & \Gamma(z)\zeta(z) + (1-2^{1-z})\Gamma'(z)\zeta(z) + (1-2^{1-z})\Gamma(z)\zeta'(z) =\\
    &= \frac{2^{1-z} \ln2 \Gamma(z)}{z-1} + 2^{1-z} \gamma \ln2 \Gamma(z) + \ln2\Gamma'(z) +
    \Big( - \frac{\ln2}{z-1} + \frac{\ln^22}{2} \Big) \Gamma(z) +o(z-1)\\
    &= \frac{2^{1-z}-1 }{z-1} \ln2 \Gamma(z) + 2^{1-z} \gamma \ln2 \Gamma(z) + \ln2\Gamma'(z) + \frac{\ln^22}{2} \Gamma(z) +o(z-1).
    \end{align}
    $$ D'où $$\begin{align}
    \int_0^{+\infty}\frac{\ln t}{1+\mathrm{e}^t}\mathrm{d}t =&-\ln^22 + \gamma \ln2 - \gamma \ln2 + \frac{\ln^22}{2} \\
    =& -\frac{\ln^22}{2}.
    \end{align}$$
  • Chaurien: Merci pour ta vigilance. Je sentais bien que ma série n'allait pas être très convergente. :-D
    Et merci pour la référence. Sandham rulez !

    L'article de 1951 mis en lien par Chaurien indique une source plus ancienne pour ce problème (1933).
  • Chaurien écrivait : http://www.les-mathematiques.net/phorum/read.php?4,2272216,2272400#msg-2272400
    [Inutile de recopier un message présent sur le forum. Un lien suffit. AD]

    Bonjour Chaurien,
    Aurais-tu un indice pour prouver l'identité au niveau MP ?
  • Ma préférence va à la méthode proposée par FdP, qui s'appuie sur deux lemmes
    $\bullet$ Lemme 1. $\displaystyle \int_{0}^{+\infty }e^{-t}\ln tdt=-\gamma \simeq -0,57722$ ;
    $\bullet$ Lemme 2. $\displaystyle \overset{+\infty }{\underset{n=1}{\sum }}(-1)^{n}\frac{\ln n}{n}=\gamma \ln
    2-\frac{1}{2}(\ln 2)^{2}\simeq 0,15987$.
    Je reviendrai tantôt sur le lemme 1, mais je réponds sans tarder sur le lemme 2, qui n'exige aucun gros théorème.

    $ \bullet $ Théorème. Soit $a\in \mathbb{N}$, et soit une fonction $f$ décroissante et positive sur $[a,+\infty \lbrack $. Pour $n\geq a$, soit $ \displaystyle S_{n}=\overset{n}{\underset{k=a}{\sum }}f(k)$ et $ \displaystyle I_{n}=\int_{a}^{n}f(t)dt$.
    Alors, quand $n\rightarrow +\infty $ : $S_{n}=I_{n}+K+o(1)$, où $K$ est une constante réelle.

    $\bullet $ Application 1. Pour $n\in \mathbb{N}^{\ast }$, soit $ \displaystyle H_{n}=\overset{n}{\underset{k=1}{\sum }}\frac{1}{k}$. Alors, quand $n\rightarrow +\infty $ : $H_{n}=\ln n+\gamma +o(1)$ quand $n\rightarrow +\infty $.
    Ce réel $\gamma $ est la constante d'Euler, et $\gamma \simeq 0,57722$.

    $\bullet $ Application 2. Pour $n\in \mathbb{N}^{\ast }$, soit $ \displaystyle W_{n}=\overset{n}{\underset{k=1}{\sum }}\frac{\ln k}{k}$. Alors, quand $%
    n\rightarrow +\infty $\ : $W_{n}=\frac{1}{2}(\ln n)^{2}+C+o(1)$.

    $\bullet $ Pour $n\in \mathbb{N}^{\ast }$, soit $\ \displaystyle T_{n}=\overset{n}{\underset{k=1}{\sum }}(-1)^{k}\frac{\ln k}{k}$.
    Alors : $ \displaystyle T_{2n}=\overset{n}{\underset{k=1}{\sum }}\frac{\ln (2k)}{2k}-\overset{n}{\underset{k=1}{\sum }}\frac{\ln (2k-1)}{2k-1} =2\overset{n}{\underset{k=1}{\sum }}\frac{\ln (2k)}{2k}-W_{2n}$
    $~~~~~~~~~~~~~~~~~ \displaystyle =\overset{n}{\underset{k=1}{\sum }}\frac{\ln 2+\ln k}{k}-W_{2n}$$ \displaystyle =(\ln 2)H_{n}+W_{n}-W_{2n}$
    $~~~~~~~~~~~~~~~~~ \displaystyle =(\ln 2)(\ln n+\gamma +o(1))+(\frac{1}{2}(\ln n)^{2}+C+o(1))-(\frac{1}{2}%
    (\ln (2n))^{2}+C+o(1))$
    $~~~~~~~~~~~~~~~~~ \displaystyle =\gamma \ln 2-\frac{1}{2}(\ln 2)^{2}+o(1)$.
    Il en résulte : $ \displaystyle \overset{+\infty }{\underset{n=1}{\sum }}(-1)^{n}\frac{\ln n}{n}=\gamma \ln 2-\frac{1%
    }{2}(\ln 2)^{2}\simeq 0,15987$.
    Il est remarquable que ce calcul de somme de série se traite par un calcul de développement limité, mais au fond c'est normal car une somme de série c'est une limite, et pour calculer une limite, quoi de mieux qu'un développement limité ?
    Je crois avoir vu ce problème dans l'American Mathematical Monthly il y a bien des années, mais je ne retrouve pas la référence.
    On le retrouve dans : RMS 109-5, janvier 1999, n° 43, oral 1998, X-MP.
    Bonne journée.
    Fr. Ch.
    07/07/2021
  • Merci à tous pour toutes ces idées. La preuve que Chaurien propose a ma préférence, je la mentionne grosso modo dans ce qui suit (elle me fait penser à la preuve permettant de calculer certaines intégrales de Cauchy-Frullani).

    Pour $a,\epsilon$ réels strictement positifs, lorsque $\epsilon$ tend vers $0$ :
    \begin{align*}
    \int_\epsilon^{+\infty}\frac{\ln(x)}{1+\mathrm{e}^{ax}}\mathrm{d}x
    &=\frac 1a\int_\epsilon^{+\infty}(-\ln(x))\frac{-a\mathrm{e}^{-ax}}{1+\mathrm{e}^{-ax}}\mathrm{d}x \\

    &\stackrel{\text{IPP}}{=} \frac{\ln(\epsilon)\ln(1+\mathrm{e}^{-a\epsilon})}a +\frac 1a\int_\epsilon^{+\infty}\frac{\ln(1-\mathrm{e}^{-2ax})}x\mathrm{d}x-\frac 1a\int_\epsilon^{+\infty}\frac{\ln(1-\mathrm{e}^{-ax})}x\mathrm{d}x \\

    &=\frac{\ln(2)\ln(\epsilon)}a+o(1)+\frac 1a\int_{2\epsilon}^{+\infty}\frac{\ln(1-\mathrm{e}^{-ax})}x\mathrm{d}x-\frac 1a\int_\epsilon^{+\infty}\frac{\ln(1-\mathrm{e}^{-ax})}x\mathrm{d}x \\

    &=\frac{\ln(2)\ln(\epsilon)}a+o(1)-\frac 1a\int_{1}^{2}\frac{\ln(1-\mathrm{e}^{-a\epsilon x})}x\mathrm{d}x\\

    &\stackrel{\text{IPP}}{=}\frac{\ln(2)\ln(\epsilon)}a+o(1)-\frac{\ln(2)\ln(1-\mathrm{e}^{-2a\epsilon})}a+\epsilon\int_{1}^{2}\frac{\ln(x)\mathrm{e}^{-a\epsilon x}}{1-\mathrm{e}^{-a\epsilon x}}\mathrm{d}x\\

    &=\frac{\ln(2)\ln(\epsilon)}a+o(1)-\frac{\ln(2)\ln(2a)}a-\frac{\ln(2)\ln(\epsilon)}a
    +\int_{1}^{2}\ln(x)\frac{\epsilon}{\mathrm{e}^{a\epsilon x}-1}\mathrm{d}x,

    \end{align*} avec $\displaystyle\frac{\epsilon}{\mathrm{e}^{a\epsilon x}-1}\leq\frac 1{ax}$.

    À la limite lorsque $\epsilon\to 0$, on obtient finalement
    $\displaystyle\int_0^{+\infty}\frac{\ln(x)}{1+\mathrm{e}^{ax}}\mathrm{d}x=-\frac{\ln(2)\ln(2a)}a+\int_{1}^{2}\frac{\ln(x)}{ax}\mathrm{d}x=
    -\frac{\ln(2)\ln(2a)}a+\frac{\ln^2(2)}{2a}$.

    [Merci AD pour la mise en forme]
    [À ton service. :-) AD]
  • Bonsoir
    Une méthode de plus pour une intégrale de plus.
    D'après
    $$\frac{\ln(t)}{1+e^t} = \frac{2\ln(t)}{1-e^{2t}} - \frac{\ln(t)}{1-e^{t}}.
    $$ Soit $0<\epsilon<\pi$
    \begin{align}
    \int_{\epsilon}^{+\infty}\frac{\ln(t)}{1+e^t} &= 2\int_{\epsilon}^{+\infty}
    \frac{\ln(t)}{1-e^{2t}}dt - \int_{\epsilon}^{+\infty} \frac{\ln(t)}{1-e^{t}}dt\nonumber\\
    &= \int_{2\epsilon}^{+\infty}
    \frac{\ln(\frac{u}{2})}{1-e^{u}}du - \int_{\epsilon}^{+\infty} \frac{\ln(t)}{1-e^{t}}dt\nonumber\\
    &= \int_{2\epsilon}^{\epsilon}
    \frac{\ln(t)}{1-e^{t}}dt - \ln(2) \int_{2\epsilon}^{+\infty} \frac{1}{1-e^{t}}dt\nonumber\\
    &= \int_{\epsilon}^{2\epsilon}
    \frac{\ln(t)}{t}\frac{t}{e^{t}-1}dt - \ln(2)\ln(1-e^{-2\epsilon})\nonumber\\
    &= \sum_{n=0}^{+\infty}\frac{B_n}{n!}\int_{\epsilon}^{2\epsilon}t^{n-1}\ln(t)dt - \ln(2)\ln(1-e^{-\epsilon}) \quad ; \qquad \frac{t}{e^{t}-1} = \sum_{n=0}^{+\infty}\frac{B_n}{n!}t^n \quad (|t|<2\pi) \nonumber\\
    &= \int_{\epsilon}^{2\epsilon} \frac{\ln(t)}{t}dt + \sum_{n=1}^{+\infty}\frac{B_n}{n!}\int_{\epsilon}^{2\epsilon}t^{n-1}\ln(t)dt - \ln(2)\ln(1-e^{-2\epsilon})\nonumber\\
    &= \left[ \frac{\ln^2(t)}{2}\right]_{\epsilon}^{2\epsilon} + \sum_{n=1}^{+\infty}\frac{B_n}{n!}\left[ \frac{\ln(t)t^n}{n} - \frac{t^n}{n^2}\right]_{\epsilon}^{2\epsilon} - \ln(2)\ln(1-e^{-2\epsilon})\nonumber\\
    &= \frac{\ln^2(2\epsilon)}{2} - \frac{\ln^2(\epsilon)}{2} - \ln(2)\ln(1-e^{-2\epsilon}) + o(\epsilon) \nonumber\\
    &= \frac{\ln^2(2)}{2} + \ln(2)\ln(\epsilon) - \ln(2)\ln(1-e^{-2\epsilon}) + o(\epsilon)\nonumber\\
    &= \frac{\ln^2(2)}{2} - \ln(2)\ln\Big(\frac{1-e^{-2\epsilon}}{\epsilon} \Big) + o(\epsilon)\nonumber.
    \end{align} Il suffit maintenant de tendre $\epsilon$ vers $0$.

    [$\LaTeX$ fournit la commande \ln qui gére police et espacements. ;-) AD]
  • @dedekind93 intéressante cette intégrale
  • Il apparaît que cette intégrale $\displaystyle I=\int_{0}^{+\infty }\frac{\ln t}{1+e^{t}}dt$ se calcule de plusieurs façons.
    Je continue sur la méthode proposée par FdP, qui repose comme j'ai dit sur les deux lemmes :
    $ \displaystyle\overset{+\infty }{\underset{n=1}{\sum }}(-1)^{n}\frac{\ln n}{n}=\gamma \ln2-\frac{1}{2}(\ln 2)^{2}$, $~~ \displaystyle \int_{0}^{+\infty }e^{-t}\ln tdt=-\gamma$, plus la somme bien connue : $ \displaystyle \overset{+\infty }{\underset{n=1}{\sum }}\frac{(-1)^{n-1}}{n}=\ln 2$.
    On fait une interversion série-intégrale, qui doit être justifiée, comme FdP l'a signalé. Comme la série en question est une série géométrique, on a l'expression explicite du reste, et l'interversion ne requiert aucun gros théorème.

    $\bullet $ On part de la somme finie de la suite géométrique : $ \displaystyle \overset{n-1}{\underset{k=0}{\sum }}(-1)^{k}e^{-kt}=\frac{1-(-1)^{n}e^{-nt}}{1+e^{-t}}$, d'où l'interversion légitime : $\displaystyle I=\int_{0}^{+\infty }\frac{%
    e^{-t}\ln t}{1+e^{-t}}dt=\overset{n-1}{\underset{k=0}{\sum }}(-1)^{k}\int_{0}^{+\infty }e^{-(k+1)t}\ln t\mathbf{~}dt+R_{n}$, avec : $\displaystyle R_{n}=(-1)^{n}\int_{0}^{+\infty }\frac{e^{-(n+1)t}\ln t}{1+e^{-t}}dt$.

    $\bullet $ On a : $\displaystyle \left\vert R_{n}\right\vert \leq \int_{0}^{+\infty}e^{-(n+1)t}\left\vert \ln t\right\vert dt
    \overset{x=(n+1)t}=\frac{1}{n+1}\int_{0}^{+\infty}e^{-x}\left\vert \ln x-\ln (n+1)\right\vert dx$
    $ ~~~~~~~~~~~~~~~~~~~~~\displaystyle\leq \frac{1}{n+1}\int_{0}^{+\infty
    }e^{-x}\left\vert \ln x\right\vert dx+\frac{\ln (n+1)}{n+1}\int_{0}^{+\infty
    }e^{-x}dx$, d'où : $\displaystyle \underset{n\rightarrow +\infty }{\lim }R_{n}=0$.

    $\bullet $ On a de plus : $\displaystyle U_{k}=\int_{0}^{+\infty }e^{-(k+1)t}\ln tdt
    \overset{x=(k+1)t}=\int_{0}^{+\infty }e^{-x}\ln \frac{x}{k+1}\cdot \frac{dx}{k+1}$$\displaystyle$
    $~~~~~~~~~~~~~~~~~~~~\displaystyle =\frac{1}{k+1}\int_{0}^{+\infty }e^{-x}\ln x\mathbf{~}dx-\frac{\ln (k+1)}{k+1}\int_{0}^{+\infty }e^{-x}dx=-\frac{\gamma}{k+1}-\frac{\ln (k+1)}{k+1}$.

    $\bullet $ Et l'on en conclut : $\displaystyle I= \overset{+\infty }{\underset{k=0}{\sum }} {(-1)^k U_k}
    \overset{h=k+1}=-\gamma \overset{+\infty }{\underset{h=1}{\sum }}\frac{(-1)^{h-1}}{h}+\overset{+\infty }{\underset{h=1}{\sum }}(-1)^{h}\frac{\ln h}{h}=-\frac{1}{2}(\ln 2)^{2}$.

    Bonne soirée.
    Fr. Ch.
    09/07/2021
  • Bonsoir Dedekind, comment par une simple majoration par 1/ax ,tu conclus la limite de cette dernière "en bleu"?, merci.124758
  • @hunter j ‘ai l’impression c’est de la convergence dominée de Lebesgue
  • @hunter** : Je pense aussi que Dedekind93 conclut par convergence dominée. Sinon, on peut observer que pour tout $x\in[1,2]$, $$\left\lvert\frac{\varepsilon}{\mathrm e^{a\varepsilon x}-1}-\frac{1}{ax}\right\rvert=\frac{1}{ax}\frac{\mathrm e^{a\varepsilon x}-1-a\varepsilon x}{\mathrm e^{a\varepsilon x}-1}=\frac{1}{ax}\frac{(a\varepsilon x)/2!+(a\varepsilon x)^2/3!+\cdots}{1+(a\varepsilon x)/2!+\cdots}$$ donc $$\left\lvert\frac{\varepsilon}{\mathrm e^{a\varepsilon x}-1}-\frac{1}{ax}\right\rvert\leq\frac{1}{ax}\bigl((a\varepsilon x)/2!+(a\varepsilon x)^2/3!+\cdots\bigr)\leq \frac{1}{ax}\bigl(\mathrm{e}^{a\varepsilon x}-1\bigr)\leq \frac{\mathrm e^{2a\varepsilon}-1}{a},$$ d'où $$\left\lvert\int_1^2\ln x\frac{\varepsilon}{\mathrm e^{a\varepsilon x}-1}\mathrm dx-\int_1^2\frac{\ln x}{ax}\mathrm dx\right\rvert\leq \frac{\mathrm e^{2a\varepsilon}-1}{a}\int_1^2\ln x\mathrm dx\xrightarrow[\varepsilon\to 0]{}0.$$
  • On a juste besoin de montrer que la fonction $x\mapsto \dfrac{1}{\text{e}^{a\epsilon x}-1}$ est bornée pour $x\in[1,2]$ et $\left|\epsilon\right|<1$. La fonction en question est monotone sauf erreur de ma part.
  • mmm, Ok c'est très intéressant d'utiliser les epsilons et le Th de CV.D dans cette intégrale, c'est vraiment une très bonne intégrale, personnellement, je n'ai pas fait auparavant quelque chose qui ressemble!..je trouve un bon goût ici ..,merci Etanche, Audeo et Fin de partie.
  • Bonjour hunter**. En posant $\displaystyle h(\epsilon,x)=\frac{\epsilon}{\mathrm{e}^{a\epsilon x}-1}$ si $\epsilon\in\mathbb{R}_+^*$ et $\displaystyle h(0,x)=\frac{1}{ax}$, il me semble que l'on peut appliquer le théorème de continuité des intégrales dépendant d'un paramètre grâce à la majoration uniforme en $\epsilon$ que j'ai donnée (ça revient au même qu'une convergence dominée bien sûr).
    Oui, c'est une intégrale que je trouve intéressante, et qui me semble plus simple que $\displaystyle\int_0^{+\infty}\ln(t)\mathrm{e}^{-t}\mathrm{d}t$ pour laquelle je ne connais pas de méthode aussi "simple" de calcul (appel aux érudits ! ;-)).
  • Pour calculer l'intégrale $\displaystyle \int_0^\infty \text{e}^{-x}\ln xdx$ il faut considérer l'intégrale $\displaystyle \int_0^1 \dfrac{1-(1-x)^n}{x}dx$
  • Bonsoir,
    $\int_0^\infty \text{e}^{-x}\ln xdx$ est la dérivée de $\Gamma$ en $1$.
  • \begin{align}J_n&=\int_0^1 \frac{1-(1-x)^n}{x}dx\\
    &\overset{y=1-x}=\int_0^1 \frac{1-y^n}{1-y}dy\\
    &=\int_0^1 \left(\sum_{k=0}^{n-1} y^k\right)dy\\
    &=\sum_{k=0}^{n-1}\int_0^1 y^kdy\\
    &=\sum_{k=0}^{n-1}\frac{1}{k+1}\\
    &=\sum_{k=1}^{n}\frac{1}{k}\\
    &=H_n
    \end{align}

    Par ailleurs,
    \begin{align}J_n&\overset{y=nx}=\int_0^n \frac{1-(1-\frac{y}{n})^n}{y}dy\\
    &=\int_0^1 \frac{1-(1-\frac{y}{n})^n}{y}dy+\int_1^n \frac{1-(1-\frac{y}{n})^n}{y}dy\\
    &=\ln n+\int_0^1 \frac{1-(1-\frac{y}{n})^n}{y}dy-\int_1^n \frac{(1-\frac{y}{n})^n}{y}dy\\
    \end{align}

    Or:
    \begin{align}\lim_{n\rightarrow \infty}\int_0^1 \frac{1-(1-\frac{y}{n})^n}{y}dy-\int_1^n \frac{(1-\frac{y}{n})^n}{y}dy=\int_0^1\frac{1-\text{e}^{-x}}{x}dx-\int_1^\infty \frac{\text{e}^{-x}}{x}dx\\
    \lim_{n\rightarrow \infty}\left(H_n-\ln n\right)=\gamma
    \end{align}
    Donc:
    \begin{align}\int_0^1\frac{1-\text{e}^{-x}}{x}dx-\int_1^\infty \frac{\text{e}^{-x}}{x}dx=\gamma\end{align}

    \begin{align}\int_0^\infty \text{e}^{-x}\ln xdx&=\underbrace{\int_0^1 \text{e}^{-x}\ln xdx}_{\text{IPP}}+\int_1^\infty \text{e}^{-x}\ln xdx\\
    &=\Big[\left(1-\text{e}^{-x}\right)\ln x\Big]_0^1-\int_0^1 \frac{1-\text{e}^{-x}}{x}dx+\int_1^\infty \text{e}^{-x}\ln xdx\\
    &=-\int_0^1 \frac{1-\text{e}^{-x}}{x}dx+\underbrace{\int_1^\infty \text{e}^{-x}\ln xdx}_{\text{IPP}}\\
    &=-\int_0^1 \frac{1-\text{e}^{-x}}{x}dx+\Big[-\text{e}^{-x}\ln x\Big]_1^{\infty}+\int_1^{\infty} \frac{\text{e}^{-x}}{x}dx\\
    &=-\int_0^1 \frac{1-\text{e}^{-x}}{x}dx+\int_1^{\infty} \frac{\text{e}^{-x}}{x}dx\\
    &=\boxed{-\gamma}
    \end{align}


    PS:
    J'ai corrigé quelques coquilles.
  • Merci L2M et merci Fdp d'avoir détaillé tout cela, avec une version la plus simple possible. Je trouve toujours cette preuve délicate ne serait-ce que par sa structure en 2 parties, $x\leq 1$ et $x\geq 1$.
  • J'ai omis la partie délicate: justifier l'utilisation d'un théorème de convergence dominée.
  • Bonjour,

    Autre possibilité pour le calcul de $I=\int_0^{+\infty}\mathrm e^{-t}\ln t\mathrm dt$ (le lemme 1 de Chaurien) suite au message de Fin de Partie : $$I=\lim_{n\to+\infty}I_n, \quad I_n:=\int_0^n\left(1-\frac tn\right)^n\ln t\mathrm dt \;;$$ par parties, avec : $$\left\{\begin{aligned}&u(t)=\ln t &&u'(t)=\frac 1t \\ &v'(t)=\left(1-\frac tn\right)^n && v(t)=\frac{n}{n+1}\biggl(1-\left(1-\dfrac tn\right)^{n+1}\,\biggr)\end{aligned}\right.$$ on a : $$I_n = \frac{n}{n+1}\Biggl(\ln n-\int_0^n\frac{1}{n}\cdot\frac{1-\left(1-t/n\right)^{n+1}}{1-\left(1-t/n\right)}\mathrm dt\Biggr) =\frac{n}{n+1}\Biggl(\ln n-\sum_{k=0}^n\int_0^n\frac{1}{n}\left(1-\frac tn\right)^k\mathrm dt\Biggr)$$ d'où $$I_n=\dfrac{n}{n+1}\left(\ln n-H_{n}-\dfrac{1}{n+1}\right)\xrightarrow[n\to+\infty]{}-\gamma.$$
  • Pour montrer que $I_n\to I$, on peut procéder comme suit : Comme $\int_n^{+\infty} \mathrm e^{-t}\ln t\mathrm dt\to 0$, il suffit de montrer que :$$D_n:=\int_0^{n} \left(\mathrm e^{-t}-\biggl(1-\frac tn\right)^n\,\biggr)\ln t\mathrm dt\xrightarrow[n\to+\infty]{}0.$$ Or, pour tout $t\in[0,n]$,$$\left(1-\frac tn\right)^n\leqslant \mathrm e^{-t}\leqslant \left(1+\frac tn\right)^{-n}$$ donc $$0\leqslant \mathrm e^{-t}-\left(1-\frac tn\right)^n\leqslant \left(1+\frac tn\right)^{-n}-\left(1-\frac tn\right)^n=\left(1+\frac tn\right)^{-n}\biggl(1-\biggl(1-\frac{t^2}{n^2}\biggr)^n\,\biggr)$$ avec, d'une part, d'après l'inégalité de Bernoulli, $$1-\biggl(1-\frac{t^2}{n^2}\biggr)^n\leq \frac{t^2}{n}$$ et, d'autre part, pour tout entier $n\geqslant 4$, $$\left(1+\frac tn\right)^n\geq 1+\binom n4\frac{t^4}{n^4}\geqslant 1+\frac{t^4}{256}.$$ Il suit, pour tout entier $n\geqslant 4$, $$\lvert D_n\rvert\leqslant \frac {1}n\int_0^{+\infty}\frac{t^2\lvert\ln t\rvert}{1+t^4/256}\mathrm dt,$$ d'où la conclusion.
  • Merci Audeo pour ce petit raccourci (la convergence de $(I_n)$ étant quand même plus rapide par convergence dominée !).
  • \begin{align}\int_0^{+\infty}\frac{\ln(x)}{1+\mathrm{e}^x}\mathrm{d}x&\overset{y=\text{e}^{-x}}=\int_0^1 \frac{\ln\left(\ln\left(\frac{1}{x}\right)\right)}{1+x}dx\end{align}
    On a le théorème suivant.
    $\displaystyle I=\int_0^1 Q(x)\ln\left(\ln\left(\frac{1}{x}\right)\right)dx$
    Si la fonction $Q$ est analytique en $x=0$ et donc au voisinage de $0$ on a $\displaystyle Q(x)=\sum_{n=0}^\infty a_nx^n$.
    On définit $\displaystyle L(s)=\sum_{n=0}^\infty \dfrac{a_n}{(n+1)^s}$.
    On a alors:
    $\displaystyle I=L^\prime(1)-\gamma L(1)$

    Dans notre cas d'espèce
    \begin{align}Q(x)=\dfrac{1}{1+x}&=\sum_{n=0}^\infty (-1)^n x^n\\
    L(s)&=\sum_{n=0}^\infty \frac{(-1)^n}{(n+1)^s}\\
    &=2^{-s}\left(2^s-2\right)\zeta(s)\\
    L^\prime(s)&=2^{-s}\left(\left(2^s-2)\zeta^\prime(s)+2\ln 2\zeta(s)\right)\right)\\
    \lim_{s\rightarrow 1} \left(2^{-s}\left(\left(2^s-2)\zeta^\prime(s)+2\ln 2\zeta(s)\right)\right)\right)&=\gamma\ln 2-\frac{\ln^2 2}{2}\\
    L(1)&=\ln 2\\
    \end{align}

    J'ai trouvé l'énoncé de ce théorème dans ce papier: Logarithmic Integrals: A Review from Gradshteyn and Ryzhik to Recent Times de Md Sarowar Morshed, février 2020.
    Méthode qu'on trouve dans un papier plus ancien :
    Integrals, an introduction to analytic number theory, Ilan Vardi, The American mathematical monthly, 95(4),1988.


    NB:
    https://www.wolframalpha.com/input/?i=limit++2^(-s)+((2^s+-+2)+ζ'(s)+++log(4)+ζ(s)),s=1
Connectez-vous ou Inscrivez-vous pour répondre.